Đến nội dung

HoangKhanh2002 nội dung

Có 461 mục bởi HoangKhanh2002 (Tìm giới hạn từ 08-05-2020)



Sắp theo                Sắp xếp  

#684910 Marathon Phương trình và hệ phương trình VMF

Đã gửi bởi HoangKhanh2002 on 18-06-2017 - 17:45 trong Phương trình - hệ phương trình - bất phương trình

Bài của Khánh sử dụng phương pháp thế, đây là 1 hệ rất quen thuộc của những đề luyện thi đại học

Từ phương trình $(2)$ $\Rightarrow y=\frac{2x^2-20x}{x^2+20}$ thế vào phương trình $(1)$ ta được 

$3x^2-9x+\frac{x(2x^3-20x)}{x^2+20}-(\frac{2x^3-20x}{x^2+20})^2-\frac{9(2x^3-20x)}{x^2+20}=0$

$\Leftrightarrow \frac{x(x-10)(x-2)(x^2-15x+20)}{(x^2+20)^2}=0$

giải phương trình tìm $x$ có vẻ bài này có 1 số nghiệm lẻ

Giải pháp này chưa hiệu quả lắm!!!

Nó sẽ không khả thi lắm trong 1 số trường hợp

Để chứng tỏ điều đó, mời chị giải 1 bài toán cùng dạng

$\boxed{\text{Bài 206}}$ Giải hệ phương trình: $\left\{\begin{matrix} 4x^3+3xy^2=7y\\ y^3+6x^2y=7 \end{matrix}\right.$




#684887 Marathon Phương trình và hệ phương trình VMF

Đã gửi bởi HoangKhanh2002 on 18-06-2017 - 14:16 trong Phương trình - hệ phương trình - bất phương trình

Bài 203: GPT $\frac{1}{\sqrt{x+3}}+\frac{1}{\sqrt{3x+1}}=\frac{2}{1+\sqrt{x}}$

Sửa lại lời giải như sau nhé (Cảm ơn tiền bối An Infinitesimal)

Áp dụng BĐT $Cauchy-Schwarz$ ta có: $\left ( \dfrac{1}{\sqrt{x+3}}+\dfrac{1}{\sqrt{3x+1}} \right )^2\leqslant 2\left ( \dfrac{1}{x+3}+\dfrac{1}{3x+1} \right )$

Mà: $\left ( \dfrac{2}{\sqrt{x}+1} \right )^2-2\left ( \dfrac{1}{x+3}+\dfrac{1}{3x+1} \right)=\dfrac{4(\sqrt{x}-1)^4}{(\sqrt{x}+1)^2(x+3)(3x+1)}\geqslant 0$

Do đó: $\frac{1}{\sqrt{x+3}}+\frac{1}{\sqrt{3x+1}}\leqslant \frac{2}{1+\sqrt{x}}$

Đẳng thức xảy ra: $\iff x=1$

Thử lại thấy thoả mãn




#684943 Marathon Phương trình và hệ phương trình VMF

Đã gửi bởi HoangKhanh2002 on 18-06-2017 - 21:22 trong Phương trình - hệ phương trình - bất phương trình

Tiếp cận hướng đẳng cấp cũng khá phức tạp.

Đây là hướng của em. Cách này đã được anh Bùi Thế Việt nghĩ ra....

 

Đã sửa

$\boxed{\text{Bài toán 204}}$ Giải hệ phương trình: $\left\{\begin{matrix} 3x^2+xy-9x-y^2-9y=0\\ 2x^3-20x-x^2y-20y=0 \end{matrix}\right.$

Bằng máy tính thử vài giá trị của $y$ rồi solve $x$, ta có được 2 nghiệm sau: $(x,y)\in \left \{ (0,0),(2,-1) \right \}\implies x+2y=0 \iff x=-2y$

Thế vào hệ ta có: $\left\{\begin{matrix} 9y(y+1)=0\\ -20y(y+1)(y-1)=0 \end{matrix}\right.$

Có ngay ý tưởng

Lấy $20(y-1)PT(1)+9PT(2)$ ta được: $20(y-1)(3x^2+xy-9x-y^2-9y)+9(2x^3-20x-x^2y-20y)=0\\ \iff (x+2y)(18x^2+15xy-60x-10y^2-80y)=0$

Trường hợp 1 dễ rồi

Trường hợp 2: Ta có hệ: $\left\{\begin{matrix} 18x^2-10y^2+15xy-60x-80y=0\\ 3x^2-y^2+xy-9x-9y=0 \end{matrix}\right.$

Hệ này cũng đã được anh Việt đưa ra dạng tổng quát

Hệ dạng: $\left\{\begin{matrix} a_{1}x^2+b_{1}y^2+c_{1}xy+d_{1}x+e_{1}y+f_{1}=0\\ a_{2}x^2+b_{2}y^2+c_{2}xy+d_{2}x+e_{2}y+f_{2}=0 \end{matrix}\right.$

Đặt: $\left\{\begin{matrix} a=a_{1}+ka_{2},b=b_{1}+kb_{2},c=c_{1}+kc_{2}\\ d=d_{1}+kd_{2},e=e_{1}+ke_{2},f=f_{1}+fa_{2} \end{matrix}\right.$

Thế vào công thức: $dec+4abf=ae^2+bd^2+fc^2$. Tìm được $k$. Lấy $PT(1)+kPT(2)$ rồi tìm $\Delta$ và phân tích nhân tử thôi

Bài toán được giải quyết khá đơn giản về mặt ý tưởng

Bài 206 tương tự dạng này, cái trong ngoặc của bài này là sử dụng tư duy đẳng cấp




#684901 Marathon Phương trình và hệ phương trình VMF

Đã gửi bởi HoangKhanh2002 on 18-06-2017 - 16:40 trong Phương trình - hệ phương trình - bất phương trình

Hình lời giải có một lỗi và lỗi đó đã hại cái lời giải hay!

 

 

Đã sửa

$\boxed{\text{Bài toán 204}}$ Giải hệ phương trình: $\left\{\begin{matrix} 3x^2+xy-9x-y^2-9y=0\\ 2x^3-20x-x^2y-20y=0 \end{matrix}\right.$




#685348 Marathon Phương trình và hệ phương trình VMF

Đã gửi bởi HoangKhanh2002 on 22-06-2017 - 10:23 trong Phương trình - hệ phương trình - bất phương trình

Tiếp lửa cho topic bằng 2 bài toán sau đây:

$\boxed{207}$ Giải phương trình $\sqrt{4x-1}+\sqrt[4]{8x-3}=4x^4-3x^2+5x$

$\boxed{208}$ Giải phương trình: $x+\sqrt{2x-1}\sqrt[3]{2x+1}=3+\sqrt{x+2}\sqrt[3]{x+4}$




#685411 Marathon Phương trình và hệ phương trình VMF

Đã gửi bởi HoangKhanh2002 on 23-06-2017 - 18:09 trong Phương trình - hệ phương trình - bất phương trình

$\boxed{208}$ Giải phương trình: $x+\sqrt{2x-1}\sqrt[3]{2x+1}=3+\sqrt{x+2}\sqrt[3]{x+4}$

Bài này có vẻ khá phức tạp nhưng có thể xử lí khá dễ như sau:

$2x-1+\sqrt{2x-1}\sqrt[3]{2x-1+2}=x+2+\sqrt{x+2}\sqrt[3]{x+2+2}$

Xét hàm: $f(t)=t+\sqrt{t}\sqrt[3]{x+2} \implies f(t)'$. Dễ dàng kiểm tra được tính đồng biến

$\implies x+2=2x-1 \implies x=3$

Thử lại và kết luận




#680672 Topic BẤT ĐẲNG THỨC ôn thi vào lớp 10 THPT 2017 - 2018

Đã gửi bởi HoangKhanh2002 on 14-05-2017 - 17:12 trong Bất đẳng thức và cực trị

Bài 3:

BĐT đã cho tương đương với: $2(\sum \sqrt{a})\geqslant 2\sum ab\Leftrightarrow 2(\sum \sqrt{a}) \geqslant (\sum a)^2-(\sum a^2) \Leftrightarrow \sum a^2+2(\sum a) \geqslant 9$

Mặt khác, theo BĐT $AM-GM$ ta có: $a^2+\sqrt{a}+\sqrt{a}\geqslant 3a\Rightarrow \sum a^2+2(\sum \sqrt{a})\geqslant 3(\sum a)=9\Rightarrow Q.E.D$

Bài 7:

Ta có: $(a+b)(b+c)(c+a)+4=(a+b+c)(ab+bc+ca)+3=\dfrac{(a+b+c)(ab+bc+ca)}{3}+\dfrac{(a+b+c)(ab+bc+ca)}{3}+\dfrac{(a+b+c)(ab+bc+ca)}{3}+3\geqslant 4\sqrt[4]{\dfrac{(a+b+c)^3(ab+bc+ca)^3}{9}}= 4(a+b+c)\sqrt[4]{\dfrac{(ab+bc+ca)^3}{9(a+b+c)}}$

Mà: $9(a+b+c)=9abc(a+b+c)\leqslant 3(ab+bc+ca)^2\leqslant (ab+bc+ca)^3$ (vì $ab+bc+ca\geqslant 3\sqrt[3]{a^2b^2c^2}=3$

Do đó ta có đpcm




#682213 Topic BẤT ĐẲNG THỨC ôn thi vào lớp 10 THPT 2017 - 2018

Đã gửi bởi HoangKhanh2002 on 28-05-2017 - 16:23 trong Bất đẳng thức và cực trị

Bài toán 90 : Cho các số thực $a,b,c$\in [0;1]$. Cmr : $a^3+b^3+c^3\leq a^3b +b^3c+c^3a$

$\boxed{\text{Lời giải bài 90}}$

BĐT đã cho tương đương với: $a^3(1-b)+b^3(1-c)+c^3(1-a)\leqslant 0$ (BĐT này luôn đúng vì $a,b,c$ $\in [0;1]$)

Dấu "=" xảy ra khi: $(a,b,c)=(1,1,1);(0,1,0);(0,0,0)(1,1,0)$ và các hoán vị




#680312 Topic ôn thi hình học vào cấp 3 chuyên

Đã gửi bởi HoangKhanh2002 on 11-05-2017 - 17:35 trong Hình học

                                                                                          :D  " Góp vui cho topic "  :D 

Mình xin tham gia topic và mở đầu bằng hai bài toán .

Bài 89 ( APMO 2000 ): Cho tam giác ABC với trung tuyến AM và phân giác AN . Đường thẳng vuông góc với AN tại N cắt AB và AM lần lượt tại P và Q . Đường thẳng vuông góc với AB tại P cắt AN tại O . Chứng minh rằng OQ vuông góc với BC . 

Bài 90 ( sưu tầm ) : Cho tam giác ABC có I là trung điểm BC , đường thẳng d đi qua I cắt AB , AC lần lượt tại M và N , đường thẳng d' đi qua I cắt AB ,AC lần lượt tại Q và P ( M và P nàm cùng phía với BC ) . MP , NQ cắt BC tại E và F . Chứng minh rằng IE = IF .

$\boxed{\text{Lời giải bài 90}}$

PS: Bài này bạn thiếu đề tí...

bai 90.png

Áp dụng định lí $Menelaus$ cho $\Delta ABC$ với $\overline{F,Q,N}$ ta có: $\frac{FB}{FC}.\frac{NC}{NA}.\frac{QA}{QB}=1\Rightarrow \frac{FB}{FC}=\frac{NA}{NC}.\frac{QB}{QA}$

$\Delta ABC$ với $\overline{E,P,M}\Rightarrow \dfrac{EC}{EB}.\dfrac{MB}{MA}.\dfrac{PA}{PC}=1\Rightarrow \dfrac{EC}{EB}=\dfrac{MA}{MB}.\dfrac{PC}{PA}$

$\Delta ABC$ với $\overline{Q,I,P}\Rightarrow \dfrac{PC}{PA}.\dfrac{QA}{QB}.\dfrac{IB}{IC}=1\Rightarrow \frac{PC}{PA}=\dfrac{QB}{QQ}$

Tương tự: $\Delta ABC$ với $\overline{M,I,N}\Rightarrow \dfrac{NC}{NA}.\dfrac{MA}{MB}.\dfrac{IB}{IC}=1\Rightarrow \dfrac{MA}{MB}=\dfrac{AN}{NC}$

Vậy $\frac{FB}{FC}=\frac{EC}{EB}\Rightarrow BF=CE\Rightarrow IE=IF$




#680598 Topic ôn thi hình học vào cấp 3 chuyên

Đã gửi bởi HoangKhanh2002 on 14-05-2017 - 00:12 trong Hình học

Bài 93(sưu tầm)

Cho tam giác ABC có góc A nhọn, nội tiếp đường tròn (O) và AB>AC. Tia phân giác cua góc BAC cắt đường tròn (O) tại D (D khác A) và cắt tiếp tuyến tại B của đường tròn (O) tại E. Gọi F la giao điểm của BD và AC.

a)Chứng minh EF//BC

b)Gọi M là giao điểm của AD va BC. Các tiếp tuyến tại B,D của đường tròn (O) cắt nhau tại N.Chứng minh rằng:$\frac{1}{BN}=\frac{1}{BE}+\frac{1}{BM}$

bai 93.png

Mặc dù đây là một bài toán dễ nhưng xuất hiện trong topic nên mình đành làm lời giải

a) Ta có: $\widehat{BEA}=\dfrac{1}{2}(sđAB-sđBD )=\dfrac{1}{2}(sđ AB-sđ DC)=\widehat{BFA}$ nên BEFA nội tiếp $\Rightarrow \widehat{BFE}=\widehat{BAE}=\widehat{DBC}\Rightarrow BC//EF$

b) Theo định lí $Thales$ ta có: $\dfrac{BN}{BM}=\dfrac{DN}{BM}=\frac{NE}{BE}\Rightarrow \dfrac{BN}{BE}+\dfrac{BN}{BM}=1\Rightarrow \dfrac{1}{BN}=\dfrac{1}{BE}+\dfrac{1}{BM}$

PS: MrCooper cho mình viết tổng hợp cùng với nhé!!!




#679996 Topic ôn thi hình học vào cấp 3 chuyên

Đã gửi bởi HoangKhanh2002 on 08-05-2017 - 19:57 trong Hình học

Bài 81(APMO):

Cho tam giác $ABC$; đường cao $AD;BE;CF$ và $O$ là tâm đường tròn ngoại tiếp tam giác $ABC$. Chứng minh rằng $OA;OB;OC;OD;OE;OF$ chia tam giác $ABC$ thành $3$ cặp tam giác bằng nhau.

attachicon.gifgeogebra-export (1).png

$\boxed{\text{Lời giải bài 81}}$

bai 81.png

Gọi P, Q lần lượt là trung điểm của BC, AC

Dễ thấy: $\widehat{BOF}=\frac{1}{2}\widehat{BOC}=\widehat{BAC}$

Lại có: $\widehat{BPO}=\widehat{BEA}=90^o \Rightarrow \Delta BOP \sim \Delta BAE (g.g)\Rightarrow \frac{OP}{EA}=\frac{OB}{BA}$

Chứng minh tương tự: $\Delta BDA \sim \Delta OQA(g.g)\Rightarrow \frac{BD}{OQ}=\frac{BA}{OA}$

Từ hai điều trên ta có: $\frac{OP}{EA}=\frac{OQ}{BD}\Rightarrow OP.BD=EA.OQ\Rightarrow S_{OBD}=S_{OAE}$

Các cặp còn lại chứng minh tương tự




#680189 Topic ôn thi hình học vào cấp 3 chuyên

Đã gửi bởi HoangKhanh2002 on 10-05-2017 - 19:19 trong Hình học

Bài 86. (IMO Shortlist 2007) Cho năm điểm A, B, C, D, E thỏa mãn ABCD là hình bình hành, BDEC là tứ giác nội tiếp đường tròn. Đường thẳng (d) qua A cắt DC và BC tại F và G thỏa mãn $EF=EG=EC$. Chứng minh rằng (d) là phân giác góc A.

$\boxed{\text{Lời giải bài 86}}$

IMO 2007.png

Gọi $H;N;K$ lần lượt là hình chiếu của $E$ xuống $DC;BC;BD$ $\Rightarrow \overline{H,N,K}$ (đường thẳng $Simson$)

Vì các $\Delta ECF; \Delta ECG$ cân nên $H;N$ lần lượt là trung điểm của $CF;CG$

Do đó: $KH//AF$$\Rightarrow K$là trung điểm của $BD\Rightarrow \overline{A,K,C}\Rightarrow \Delta EDB$ cân tại $E$

Từ đó: $\Rightarrow \widehat{BCE}=\widehat{BDE}=\widehat{EBD}=\widehat{ECF}\Rightarrow \widehat{EGC}=\widehat{EFC}$. Mà: $\widehat{EFG}=\widehat{EGF}\Rightarrow \widehat{CFG}=\widehat{CGF}\Rightarrow \widehat{DAF}=\widehat{FAB}$

Vậy bài toán được chứng minh




#650637 Topic yêu cầu tài liệu THCS

Đã gửi bởi HoangKhanh2002 on 21-08-2016 - 14:33 trong Tài liệu - Đề thi

 

 

Anh ơi file bị lỗi rồi...

Ai có thể cho em xin file khác của quyển Hình học không ạ??

Plssss...

Em cảm ơn. 

Bạn có hai quyển của NGUyễn Vũ Thanh đó chưa? Có cho mình xin link hoặc phiền bạn gửi cho mình qua địa chỉ: [email protected]. cảm ơn bạn nhiều!!!!!!!!!!!!




#678065 Topic yêu cầu tài liệu THCS

Đã gửi bởi HoangKhanh2002 on 19-04-2017 - 22:11 trong Tài liệu - Đề thi

ai co tai lieu ve he pt ko cho em xem voi 

Có đấy. Mail bạn là gì mình gửi cho. Tài liệu cực hay mình sưu tầm được




#690784 [CHUYÊN ĐỀ] CHỨNG MINH BẤT ĐẲNG THỨC

Đã gửi bởi HoangKhanh2002 on 17-08-2017 - 20:03 trong Bất đẳng thức và cực trị

       Cho các số thực dương a, b,c. Chứng minh rằng :

               $\frac{2(a^{3}+b^{3}+c^{3})}{abc}+\frac{9(a+b+c)^{2}}{a^{2}+b^{2}+c^{2}}\geqslant 33$

Dùng S.O.S như sau:

BĐT cần chứng minh tương đương với:

$\dfrac{2(a^3+b^3+c^3)}{abc}-6+\dfrac{9(a+b+c)^2}{a^2+b^2+c^2}-27\geqslant 0\\ \iff \dfrac{2(a+b+c)(a^2+b^2+c^2-ab-bc-ca)}{abc}+\dfrac{18(ab+bc+ca-a^2-b^2-c^2)}{a^2+b^2+c^2}\geqslant 0\\\iff \left [ (a-b)^2+(b-c)^2+(c-a)^2 \right ]\left ( \dfrac{a+b+c}{abc}-\dfrac{9}{a^2+b^2+c^2} \right )\geqslant 0$

Mà: $\dfrac{a+b+c}{abc}-\dfrac{9}{a^2+b^2+c^2}=\dfrac{1}{ab}+\dfrac{1}{bc}+\dfrac{1}{ca}-\dfrac{9}{a^2+b^2+c^2}\\\geqslant \dfrac{9}{ab+bc+ca}-\dfrac{9}{a^2+b^2+c^2}\geqslant 0$

Do đó: $\implies Q.E.D$




#666729 $\boxed{Topic}$ ÔN THI VÀO THPT CHUYÊN TOÁN NĂM HỌC...

Đã gửi bởi HoangKhanh2002 on 02-01-2017 - 22:29 trong Tài liệu - Đề thi

$\boxed{5}$

Dễ thấy $x,y,z>0.$

Trừ lần lượt phương trình thứ nhất và phương trình thứ hai, phương trình thứ hai và phương trình thứ ba ta được hệ mới:

$\left\{\begin{matrix} (x-y)(x+y)(x^2+y^2)+2(z-y)=0 \;\;\;\ (1) \\ (y-z)(y+z)(y^2+z^2)+2(x-z)=0 \;\;\;\ (2) \end{matrix}\right.$

Giả sử $x\geq y>0,$ từ $(1)$ suy ra $y\geq z$ suy ra $z\geq x$ (do $(2)$ ). Do đó $x\geq y\geq z\geq x>0\Leftrightarrow x=y=z$

Thay vào được $2x^4-4x+1=0$ (Sao nghiệm không đẹp nhỉ? :( )

Tiếp tục nhé: $2x^4-x+1=0\Leftrightarrow 2x^4+4x^2+2-4x^2-4x-1=0\Leftrightarrow 2(x^2+1)^2=(2x+1)^2\Leftrightarrow \sqrt{2}(x^2+1)=2x+1\vee \sqrt{2}(x^2+1)=-2x-1$

+) $\sqrt{2}(x^2+1)=-2x-1\Leftrightarrow \sqrt{2}x^2+2x+1+\sqrt{2}$ (vô nghiệm)

+) $\sqrt{2}(x^2+1)=2x+1\Leftrightarrow \sqrt{2}x^2-2x-1+\sqrt{2}\Rightarrow \Delta = 4\sqrt{2}-4\Rightarrow x=\frac{2\pm 2\sqrt{\sqrt{2}-1}}{2\sqrt{2}}=\frac{1\pm \sqrt{\sqrt{2}-1}}{\sqrt{2}}=y=z$




#666454 $\boxed{Topic}$ ÔN THI VÀO THPT CHUYÊN TOÁN NĂM HỌC...

Đã gửi bởi HoangKhanh2002 on 01-01-2017 - 14:39 trong Tài liệu - Đề thi

$\boxed{2}$ Giải hệ phương trình: $\left\{\begin{matrix} x^3+y^3+x^2(y+z)=xyz+14\\ y^3+z^3+y^2(z+x)=xyz-21\\ z^3+x^3+z^2(x+y)=xyz+7 \end{matrix}\right.$

$\boxed{3}$ Giải hệ phương trình: $\left\{\begin{matrix} (x+3)^3=3-2y\\ z^2+4y^2=8y\\ (2z-x)(x+3)=5x+16\\ z\geq 0 \end{matrix}\right.$




#668838 $\boxed{Topic}$ ÔN THI VÀO THPT CHUYÊN TOÁN NĂM HỌC...

Đã gửi bởi HoangKhanh2002 on 19-01-2017 - 12:19 trong Tài liệu - Đề thi

HỆ PHƯƠNG TRÌNH VÔ TỈ

Trở lại với topic sau khoảng thời gian dài

$\boxed{1}$ Giải hệ phương trình: $\left\{\begin{matrix} \frac{x-y\sqrt{x^2-y^2}}{\sqrt{1-x^2+y^2}}=a\\ \frac{y-x\sqrt{x^2-y^2}}{\sqrt{1-x^2+y^2}}=b \end{matrix}\right.$ với a, b là tham số

$\boxed{2}$ Giải hệ phương trình: $\left\{\begin{matrix} \frac{1}{\sqrt{x}}+\sqrt{2-\frac{1}{y}}=2\\ \frac{1}{\sqrt{y}}+\sqrt{2-\frac{1}{x}}=2 \end{matrix}\right.$




#666455 $\boxed{Topic}$ ÔN THI VÀO THPT CHUYÊN TOÁN NĂM HỌC...

Đã gửi bởi HoangKhanh2002 on 01-01-2017 - 14:49 trong Tài liệu - Đề thi

Từ $HPT$ suy ra: $\left\{\begin{matrix} x^3=13xy-12 \;\;\;\ (1) \\ y^3=-21xy+22 \;\;\;\ (2) \end{matrix}\right.$

Đặt $t=xy\Rightarrow t^3=(13t-12)(-21t+22)\Leftrightarrow t^3+273t^2-538t+{\color{Red} (264)}=0\Leftrightarrow (t-1)(t^2+274t-264)=0$

Tới đây tìm được các giá trị của $t$ hay chính là $xy$ từ đó thay vào $(1)$ và $(2)$ tìm được $x,y.$

Đáp số:

attachicon.gifds.JPG

Bạn làm đúng rồi đấy, tiếp tục đi!!! Chỉ thiếu một chút (đánh thiếu)




#672435 $\boxed{Topic}$ ÔN THI VÀO THPT CHUYÊN TOÁN NĂM HỌC...

Đã gửi bởi HoangKhanh2002 on 22-02-2017 - 22:36 trong Tài liệu - Đề thi

Trong quá trình chuẩn bị cho phần tiếp theo là "SỐ HỌC" thì mình đem ra 1 đề thi thử để các bạn cung làm

TRƯỜNG THCS ĐỒNG LẠNG-1.jpg




#680594 $\boxed{Topic}$ ÔN THI VÀO THPT CHUYÊN TOÁN NĂM HỌC...

Đã gửi bởi HoangKhanh2002 on 13-05-2017 - 23:51 trong Tài liệu - Đề thi

Mình khởi đầu bằng hai bài

$\boxed{1}$ Chứng minh rằng trong 5 số mà trong mỗi số chỉ có ước nguyên tố là 3 và 5 tồn tại hai số mà tích của chúng là số chính phương

$\boxed{2}$ Cho một hình chữ nhật 5 x m gồm các ô vuông 1 x 1. Cho biết rằng có thể đặt các quân domino kích thước 1 x 2 vào hình chữ nhật sao cho quân domino phủ kín hình chữ nhật và trong hai ô 1 x 1 của quân domino; có một ô chứa dấu $(+)$ và một ô chứa dấu $(-)$; tích của  dấu trong các ô trong cùng một hàng và trong cùng một cột đều dương.

$\boxed{\text{Lời giải bài 1}}$

5 số đó có dạng: $3^x5^y$. Vì các số này chỉ có ước nguyên tố là 3 và 5. Khi đó bộ $(x;y)$ chỉ thuộc tập: $\left \{ (chẵn;chẵn);(chẵn;lẻ);(lẻ;chẵn);(lẻ;lẻ) \right \}$

Do có 5 số phân biệt nên có 5 cặp $(x;y)$ nguyên, phân biệt. Theo nguyên lí $Dirichlet$, tồn tại 2 số cùng một phần tử của tập nên tích của 2 số này là 1 số chính phương

PS: Xin lỗi các bạn nhiều vì trong thời gian qua, mình không ghé qua topic. Lí do là dạo này mình đang phải ôn luyện Văn và Anh chuẩn bị thi HK cũng như TS. Văn quá kém...

Hôm trước mình làm đáng lẽ phải kí hiệu cho nó để mọi người không hiểu lầm. Sorry




#675821 $\boxed{Topic}$ ÔN THI VÀO THPT CHUYÊN TOÁN NĂM HỌC...

Đã gửi bởi HoangKhanh2002 on 31-03-2017 - 18:45 trong Tài liệu - Đề thi

$\boxed{16}$ Cho $a, b, c$ là các số thực dương. Tìm giá trị nhỏ nhất của biểu thức

                     $P=\frac{a^{2}}{(a+b)^{2}}+\frac{b^{2}}{(b+c)^{2}}+\frac{c}{4a}$.

$\boxed{17}$ Giải phương trình $\sqrt{5-3x}+\sqrt{x+1}=\sqrt{3x^{2}-4x+4}$.

$\boxed{16}$

Ta có: $P=\frac{a^{2}}{(a+b)^{2}}+\frac{b^{2}}{(b+c)^{2}}+\frac{c}{4a}=\frac{1}{\left ( 1+\frac{b}{a} \right )^2}+\frac{1}{\left ( 1+\frac{c}{b} \right )^2}+\frac{1}{4}.\frac{b}{a}.\frac{c}{b}\geq \frac{1}{1+\frac{b}{a}.\frac{c}{b}}+\frac{1}{4}.\frac{b}{a}.\frac{c}{b}=\left ( \frac{\frac{b}{a}.\frac{c}{b}+1}{4}+\frac{1}{1+\frac{b}{a}.\frac{c}{b}} \right )-\frac{1}{4}\geq 1-\frac{1}{4}=\frac{3}{4}$

Vì có BĐT: $\frac{1}{(1+a)^2}+\frac{1}{(1+b)^2}\geq \frac{1}{1+ab}$

$\boxed{17}$

ĐK: $-1\leq x\leq \frac{5}{3}$

Phương trình đã cho tương đương với: $6-2x+2\sqrt{(5-3x)(x+1)}=3x^2-4x+4\Leftrightarrow -3x^2+2x+2\sqrt{-3x^2+2x+5}=0\Leftrightarrow (-3x^2+2x+5)+2\sqrt{-3x^2+2x+5}-3=0\Leftrightarrow \begin{bmatrix} \sqrt{-3x^2+2x+5}=-3(KTM)\\ \sqrt{-3x^2+2x+5}=1\Rightarrow x=\frac{1\pm \sqrt{13}}{3}(TM) \end{bmatrix}$

PS: Dạo này bận quá trời.... Không có thời gian...Giờ đi học cái đã....




#675291 $\boxed{Topic}$ ÔN THI VÀO THPT CHUYÊN TOÁN NĂM HỌC...

Đã gửi bởi HoangKhanh2002 on 25-03-2017 - 17:34 trong Tài liệu - Đề thi

$\boxed{1}$ Giải phương trình: $5x^2+8x-14=(x^2+7x-10)\sqrt{x+2}$

Lời giải cho bài $\boxed{1}$

ĐK: $x\geq -2$

Phương trình đã cho tương đương với: $-(-5\sqrt{x+2}+x+7)(x\sqrt{x+2}+2)=0\Leftrightarrow \begin{bmatrix} x+7=5\sqrt{x+2}\\ x\sqrt{x+2}=-2 \end{bmatrix}\Rightarrow \begin{bmatrix} x^2-11x-1=0\\ x^3+2x^2+8=0 \end{bmatrix}\Rightarrow x=\frac{11\pm 5\sqrt{5}}{2}$




#675289 $\boxed{Topic}$ ÔN THI VÀO THPT CHUYÊN TOÁN NĂM HỌC...

Đã gửi bởi HoangKhanh2002 on 25-03-2017 - 17:22 trong Tài liệu - Đề thi

BÀI 5 : Giải phương trình:

$-4x^3+18x^2-22x+5+\frac{1}{x^2}+\frac{6x+2}{\sqrt{(5-2x)(2x-1)}}+\frac{3x^2-x+2}{\sqrt{x-1}}=6\sqrt[3]{3}$ 

BÀI 6: (Học sinh giỏi tỉnh Hà Nam 2013-2014) 

Cho các số thực dương $a,b,c$ thỏa mãn: $a^2+2b^2+3c^2=3abc$

Tìm GTNN của biểu thức: $P = 3a+2b+c+\frac{8}{a}+\frac{6}{b}+\frac{4}{c}$

 

 

$\boxed{5}$Có sai đề gì không bạn (không ra nghiệm)
 

$\boxed{6}$

Áp dụng BĐT AM - GM ta có: $3abc=a^2+2b^2+3c^2=a^2+b^2+b^2+c^2+c^2+c^2\geq 6\sqrt[6]{a^2b^4c^6}\Rightarrow 3\sqrt[6]{a^6b^6c^6}\geq 6\sqrt[6]{a^2b^4c^6}\Rightarrow \sqrt[6]{a^4b^2}\geq 2\Rightarrow 2\leq \sqrt[3]{a^2b}\leq \frac{a+a+b}{3}=\frac{2a+b}{3}\Rightarrow 2a+b\geq 6$

Ta có: $P = 3a+2b+c+\frac{8}{a}+\frac{6}{b}+\frac{4}{c}=2a+\frac{8}{a}+\frac{3}{2}b+\frac{6}{b}+c+\frac{4}{c}+\frac{1}{2}(2a+b)\geq 2.4+2.3+2.2+\frac{1}{2}.6=21$

Dấu "=" xảy ra khi: a = b = c = 2




#670978 $\boxed{Topic}$ ÔN THI VÀO THPT CHUYÊN TOÁN NĂM HỌC...

Đã gửi bởi HoangKhanh2002 on 10-02-2017 - 12:25 trong Tài liệu - Đề thi

Để topic có thể tiếp cận với nhiều bạn có trình độ khá - giỏi. Mình sẽ giảm độ khó xuống để phù hợp

$\boxed{4}$ Giải hệ phương trình: $\left\{\begin{matrix} xy+x+y=x^2-2y^2\\ x\sqrt{2y}-y\sqrt{x-1}=2x-2y \end{matrix}\right.$

$\boxed{5}$ Giải hệ phương trình: $\left\{\begin{matrix} x+\sqrt{y^2-x^2}=12-y\\ x\sqrt{y^2-x^2}=12 \end{matrix}\right.$